Complex Conjugate Inequality Proof

In summary, the conversation discusses the attempt to solve a problem involving complex numbers and an inequality. The initial approach of splitting the problem due to symmetry is deemed to be incorrect, and a new approach using polar representation is suggested. The conversation ends with a verification of the proposed inequality using elementary algebra, trigonometry, and calculus. However, further discussion is limited due to forum rules.
  • #1
dumbdumNotSmart
41
3

Homework Statement


$$
\left | \frac{z}{\left | z \right |} + \frac{w}{\left | w \right |} \right |\left ( \left | z \right | +\left | w \right | \right )\leq 2\left | z+w \right |
$$
Where z and w are complex numbers not equal to zero.
2.$$\frac{z}{\left | z \right | ^{2}}=\frac{1}{\bar{z}}$$

The Attempt at a Solution


EDIT: INCORRECT WORK[/B]
Alright, the following was my approach. I first tried to split up the problem due to symmetry.
$$
\left | \frac{z}{\left | z \right |} + \frac{w}{\left | w \right |} \right | \left | z \right | \leq \left | z+w \right |
$$
$$
\left | \frac{1}{\bar{z}} + \frac{1}{\bar{w}} \right | \left | z \right | \leq \left | z+w \right |

$$
$$
\left | \frac{\bar{z}+\bar{w}}{\bar{z}\bar{w}} \right | \left | z \right | \leq \left | z+w \right |
$$
And I was left with the following
$$
\left | \bar{z}+\bar{w} \right | \left | z \right | \leq \left | z+w \right | |
\bar{z}\bar{w}|
$$

I feel like I'm either close, or I've worked the problem entirely the wrong way including conjugates.
 
Last edited:
Physics news on Phys.org
  • #2
I don't think you can split the problem by symmetry in such a manner. I would reconsider that step. If the one term is equal to a where a<1, it means its symmetric pair just needs to be less than 2-a. If you could prove the one term is less than one, it would guarantee the sum is less than 2, but that is probably not the case. Most likely, the single term is not necessarily less than one.
 
  • Like
Likes dumbdumNotSmart
  • #3
Charles Link said:
I don't think you can split the problem by symmetry in such a manner. I would reconsider that step. If the one term is equal to a where a<1, it means its symmetric pair just needs to be less than 2-a. If you could prove the one term is less than one, it would guarantee the sum is less than 2, but that is probably not the case. Most likely, the single term is not necessarily less than one.
Hmm, yes I see what you mean. I'm really running out of ideas on how to operate. I guess I could try expanding the binomials, but that looks like a long and arduous task (one I am willing to do).
 
  • #4
dumbdumNotSmart said:
Hmm, yes I see what you mean. I'm really running out of ideas on how to operate. I guess I could try expanding the binomials, but that looks like a long and arduous task (one I am willing to do).
Suggest you square both sides because |w+z| gives you a square root. Incidentally in your second line in post 1, that should be ## |z|^2 ## in the denominator.
 
  • Like
Likes dumbdumNotSmart
  • #5
Charles Link said:
Suggest you square both sides because |w+z| gives you a square root. Incidentally in your second line in post 1, that should be ## |z|^2 ## in the denominator.
Yup, looks like I goofed. It does not change anything though does it? Symmetry was no good...
 
  • #6
dumbdumNotSmart said:
Yup, looks like I goofed. It does not change anything though does it? Symmetry was no good...
It looks like a complete expansion with squaring is your best approach. I expect terms will cancel on both sides. It is possible you will need to square a second time after that, but I do think this one will not be as difficult as it looks.
 
  • #7
dumbdumNotSmart said:

Homework Statement


$$
\left | \frac{z}{\left | z \right |} + \frac{w}{\left | w \right |} \right |\left ( \left | z \right | +\left | w \right | \right )\leq 2\left | z+w \right |
$$
Where z and w are complex numbers not equal to zero.
2.$$\frac{z}{\left | z \right |}=\frac{1}{\bar{z}}$$

The Attempt at a Solution


Alright, the following was my approach. I first tried to split up the problem due to symmetry.
$$
\left | \frac{z}{\left | z \right |} + \frac{w}{\left | w \right |} \right | \left | z \right | \leq \left | z+w \right |
$$
$$
\left | \frac{1}{\bar{z}} + \frac{1}{\bar{w}} \right | \left | z \right | \leq \left | z+w \right |

$$
$$
\left | \frac{\bar{z}+\bar{w}}{\bar{z}\bar{w}} \right | \left | z \right | \leq \left | z+w \right |
$$
And I was left with the following
$$
\left | \bar{z}+\bar{w} \right | \left | z \right | \leq \left | z+w \right | |
\bar{z}\bar{w}|
$$

I feel like I'm either close, or I've worked the problem entirely the wrong way including conjugates.

If you write ##z = r_1 e^{i \theta_1}## and ##w = r_2 e^{i \theta_2}## the left and right-hand sides of your proposed inequality become quite simple functions of ##r_1, r_2, \theta_1, \theta_2##. You can use trigonometric identities and a bit of algebra to get a relatively simple inequality to verify.
 
  • Like
Likes dumbdumNotSmart and Charles Link
  • #8
Ray Vickson said:
If you write ##z = r_1 e^{i \theta_1}## and ##w = r_2 e^{i \theta_2}## the left and right-hand sides of your proposed inequality become quite simple functions of ##r_1, r_2, \theta_1, \theta_2##. You can use trigonometric identities and a bit of algebra to get a relatively simple inequality to verify.

By proposed inequality you mean the initial inequality we are trying to prove, right? The work leading up to the bottom bit is incorrect.
 
  • #9
dumbdumNotSmart said:
By proposed inequality you mean the initial inequality we are trying to prove, right? The work leading up to the bottom bit is incorrect.

Yes, the proposed inequality is the one you are trying to prove.

When you say the work leading up to the bottom bit is incorrect, whose bottom bit are you referring to? What message number are you citing?

By using the polar representation I was able to verify your proposed inequality using only elementary algebra, trigonometry and (at one point) elementary calculus. It takes a bit of work, but it really does go through. However, I would not be comfortable with saying much more at this stage, due to PF rules, etc.
 
  • #10
Ray Vickson said:
Yes, the proposed inequality is the one you are trying to prove.

When you say the work leading up to the bottom bit is incorrect, whose bottom bit are you referring to? What message number are you citing?

By using the polar representation I was able to verify your proposed inequality using only elementary algebra, trigonometry and (at one point) elementary calculus. It takes a bit of work, but it really does go through. However, I would not be comfortable with saying much more at this stage, due to PF rules, etc.
So I went ahead and did the problem yesterday, I believe it ended well. Didn't prove one part though, when I divide both sides by double sines and cosines, I don't check if it's less than 0. https://drive.google.com/open?id=0BxlGAHNyMIneeUpXWEluZWF0TEE
 
  • #11
dumbdumNotSmart said:
So I went ahead and did the problem yesterday, I believe it ended well. Didn't prove one part though, when I divide both sides by double sines and cosines, I don't check if it's less than 1. https://drive.google.com/open?id=0BxlGAHNyMIneeUpXWEluZWF0TEE
(Explanations are in spanish, though my guess is you guys will figure it out)
 
  • #12
dumbdumNotSmart said:
(Explanations are in spanish, though my guess is you guys will figure it out)
Your ## cos(\theta)cos(\phi)-sin(\theta)sin(\phi) = cos(\theta +\phi) ## so it necessarily has absolute value less than ((edited:) or equal to )1 and won't change the sign of your inequality.
 
Last edited:
  • Like
Likes dumbdumNotSmart
  • #13
Charles Link said:
Your ## cos(\theta)cos(\phi)-sin(\theta)sin(\phi) = cos(\theta +\phi) ## so it necessarily has absolute value less than 1 and won't change the sign of your inequality.
True, I think I'll add it to my proof. Thanks Charles (Credit where due)
 
  • Like
Likes Charles Link
  • #14
dumbdumNotSmart said:
So I went ahead and did the problem yesterday, I believe it ended well. Didn't prove one part though, when I divide both sides by double sines and cosines, I don't check if it's less than 0. https://drive.google.com/open?id=0BxlGAHNyMIneeUpXWEluZWF0TEE
Here is what I did: if ##w = re^{i \lambda}## and ##z = k r e^{i \phi}## (##r,k > 0##) then
$$L \equiv \left| \frac{w}{|w|} + \frac{z}{|z|} \right| (|w|+|z|) = r(1+k) |e^{i \lambda} + e^{ i \phi}|$$
and
$$R \equiv 2 |w+z| = 2 |r e^{i \lambda} + rk e^{i \phi}|$$.
Without loss of generality we can assume ##\lambda = 0##; if not, just use the fact that
$$ L = |e^{-i\lambda}|L= r(1+k) |e^{-i \lambda}(e^{i \lambda} + e^{ i \phi})| $$
and
$$ R = |e^{- i \lambda}| R = 2 |e^{-i \lambda}(r e^{i \lambda} + rk e^{i \phi})|$$
use the new variables ##w = r## and ##z = k r e^{i \theta}##, where ##\theta = \phi - \lambda##.

We have
$$L = r (1+k) \sqrt{2(1+c)},\; R = 2 r \sqrt{k^2+1+2 k c},$$
where ##c = \cos(\theta)##. Thus,
$$ \begin{array}{rcl}R-L &=& r [ 2 \sqrt{1+k^2 + 2kc} - (1+k) \sqrt{2(1+c)}] \\
&=& r \sqrt{1+c}\, f(k,c),
\end{array}$$
where
$$f(k,c) = 2 \sqrt{ 2k + (k-1)^2/(1+c)} - \sqrt{2} (k+1) $$
(because ##1+k^2+2kc = (k-1)^2 + 2k(1+c)##).

For any fixed ##k > 0## the function ##f(k,c)## is strictly decreasing in ##c## on the interval ##-1 < c \leq 1##, so for ##-1 < c < 1## we have ##f(k,c) > f(k,1) = 0##. Thus, ##R-L > 0## for ##-1 \leq c < 1##, and ##R=L## for ##c = 1 \; (\theta = 0)##.
 
  • Like
Likes dumbdumNotSmart

What is the "Complex Conjugate Inequality Proof"?

The complex conjugate inequality proof is a mathematical proof that involves manipulating complex numbers and their conjugates to prove an inequality statement.

What is the purpose of using complex conjugates in an inequality proof?

The use of complex conjugates in an inequality proof allows for easier manipulation of complex numbers and simplification of the inequality statement.

What are the steps involved in a complex conjugate inequality proof?

The steps involved in a complex conjugate inequality proof include writing the inequality statement in terms of complex numbers, multiplying both sides by the complex conjugate, rearranging the terms, and using the properties of complex numbers to simplify the statement.

How do you know when to use a complex conjugate in an inequality proof?

A complex conjugate should be used when the inequality statement involves complex numbers and the goal is to simplify and prove the statement using algebraic methods.

What are some common mistakes made when using complex conjugates in an inequality proof?

Common mistakes when using complex conjugates in an inequality proof include forgetting to multiply both sides by the complex conjugate, making errors in the rearranging and simplification steps, and not fully understanding the properties of complex numbers.

Similar threads

  • Calculus and Beyond Homework Help
Replies
8
Views
2K
  • Calculus and Beyond Homework Help
Replies
3
Views
1K
Replies
2
Views
1K
  • Calculus and Beyond Homework Help
Replies
17
Views
1K
  • Calculus and Beyond Homework Help
Replies
6
Views
484
  • Calculus and Beyond Homework Help
Replies
16
Views
1K
  • Calculus and Beyond Homework Help
Replies
4
Views
976
  • Calculus and Beyond Homework Help
Replies
2
Views
716
  • Calculus and Beyond Homework Help
Replies
10
Views
1K
  • Calculus and Beyond Homework Help
Replies
9
Views
1K
Back
Top